Preptest 37-2-19 Forum

Prepare for the LSAT or discuss it with others in this forum.
Post Reply
User avatar
ltowns1

Silver
Posts: 717
Joined: Mon May 26, 2014 1:13 am

Preptest 37-2-19

Post by ltowns1 » Tue Aug 11, 2015 9:39 pm

Hey can't we treat this as almost a sufficient assumption question with the conditionals that it gives? (I know it's an necessary assumption question)


Traditional self help ----> helps people quit smoking??

joewoo198256

New
Posts: 12
Joined: Mon Aug 10, 2009 10:28 am

Re: Preptest 37-2-19

Post by joewoo198256 » Wed Aug 19, 2015 11:01 pm

I don't think so. it is just a necessary assumption because we can use negation test to verify its usefulness. If it is a sufficient assumption, then even if it fails the negation test that does not mean the argument cannot hold.

Post Reply

Return to “LSAT Prep and Discussion Forum”